Author Topic: EM Drive Developments - related to space flight applications - Thread 3  (Read 3130562 times)

Offline wallofwolfstreet

  • Full Member
  • *
  • Posts: 165
  • Liked: 169
  • Likes Given: 436

I think that the issue with closed systems is the reason that Paul March proposed a higher-dimensional space mechanism. What looks like a closed system in our 3+1 dimensional everyday reality, could easily be an open system in 5D+ spacetime. And this is why the experimentational factor is so vital. We need a demonstrator with high enough unidirectional force outout that can be reliably reproduced by any random, sufficiently technically skilled person on this planet. As string theories predict, reality is actually a lot higher-dimensional than what we can perceive and conceive with these little brains of ours.
:)

There just doesn't seem to be any way around it in my mind.  Perhaps this is the first experimental evidence of a 5D spacetime (or interacting with the QV) - otherwise the thrust is an experimental artifact. I just don't see how it can be any other way. I do 'believe' there is thrust and it's not an experimental artifact - I also believe the theories proposed by Shawyer and Yang are wrong and perhaps White is on the right track - Shawyer's and Yang's drives just happen to work based on entirely different principles than they proposed.

White is definitely NOT on the right track. Yang is closer than anyone and simply didn't disclose the details, and used a poorly constructed example to explain it, but her force equation is correct "IFF" you plug in the correct field amplitudes AND phases.

Simply put, it moves because inside there is an EM stress energy tensor that is skewed to one side by constructive and destructive interference with the source. It would have a lot more thrust if the balance were 50/50, but as it is, only a small percentage of the waves walk out of phase to cause the interference. So the standing waves dominate, and there is very little thrust. The percentage results in higher NET thrust for higher Q, but it would be a more efficient thruster if the percentage were higher and the Q were lower.

Todd

Todd, to maybe help with my understanding of your theory, is there anyway that you could succinctly say what is it within your theory that causes COM to be obeyed?  We know that the EMdrive has a net force, and subsequent gain in physical momentum, in one direction. 

What (matter, EM waves, etc.) is it that moves in the opposite direction such that COM holds? 

Simply put, for me at least, talking about a "skewed EM stress energy tensor" and constructive vs destructive interference doesn't offer any insight into the conservation of momentum issue because those words don't have any meaning for me in terms of momentum.

Offline Star One

  • Senior Member
  • *****
  • Posts: 13996
  • UK
  • Liked: 3974
  • Likes Given: 220
In conventional physics, it is clear that the momentum of the radiation injected into the cavity produces a back reaction upon the source of that radiation. Therefore if you put a box around [source + cavity], where the source includes a portable power supply (e.g. battery) - then the net momentum of the system is zero forever as viewed from outside the box.

If there is a radiation leak, thrust no greater than P/c ("equivalent photon rocket") may result from such a system, including thermal effects originating from the input power P.

Clearly here the claim being made is not conventional physics because:

1. The claim is made that the box as a whole will move
and
2. The claim is made that the magnitude of the thrust causing this movement exceeds the maximum expected thrust of the equivalent photon rocket by orders of magnitude.

Thus any successful attempt at explanation will not use conventional physics.


There is a corollary to all this:

If you attempt to explain this effect with conventional physics alone, you have made an error.

Thank you, thank  you, thank you.  Exactly.

But we've covered all of this literally dozens of times up thread.  Pretty please, don't rehash what we've already covered ad nauseum, and instead focus on what we haven't covered so that the thread doesn't loose meaning or focus.  Every few weeks this thread has a collective breather when someone yells "Unicorns don't exist!" then the lurkers ask a bunch of newbie questions, and then we go back to researching while the audience listens quietly. :)

But is that a closed or open system.

Face the unfortunate fact that until more data is received from experiments conducted at a high level this thread could be doomed to go in circles.:(

We'll just have to hope that maybe in the next six months fresh data of a qualitative standard is released.
« Last Edit: 06/02/2015 08:13 pm by Star One »

Offline sghill

  • Full Member
  • ****
  • Posts: 1682
  • United States
  • Liked: 2092
  • Likes Given: 3200
But is that a closed or open system.

Face the unfortunate fact that until more data is received from experiments conducted at a high level this thread could be doomed to go in circles.

Yes, that can be a consequence, or you can jump in with the experimenters and focus on the experiments in the hope that more experimental results will break the rhetoric cycle.
Bring the thunder!

Offline WarpTech

  • Full Member
  • ****
  • Posts: 1407
  • Do it!
  • Statesville, NC
  • Liked: 1453
  • Likes Given: 1925
...

White is definitely NOT on the right track. Yang is closer than anyone and simply didn't disclose the details, and used a poorly constructed example to explain it, but her force equation is correct "IFF" you plug in the correct field amplitudes AND phases.

Simply put, it moves because inside there is an EM stress energy tensor that is skewed to one side by constructive and destructive interference with the source. It would have a lot more thrust if the balance were 50/50, but as it is, only a small percentage of the waves walk out of phase to cause the interference. So the standing waves dominate, and there is very little thrust. The percentage results in higher NET thrust for higher Q, but it would be a more efficient thruster if the percentage were higher and the Q were lower.

Todd

Todd, to maybe help with my understanding of your theory, is there anyway that you could succinctly say what is it within your theory that causes COM to be obeyed?  We know that the EMdrive has a net force, and subsequent gain in physical momentum, in one direction. 

What (matter, EM waves, etc.) is it that moves in the opposite direction such that COM holds? 

Simply put, for me at least, talking about a "skewed EM stress energy tensor" and constructive vs destructive interference doesn't offer any insight into the conservation of momentum issue because those words don't have any meaning for me in terms of momentum.

Before I try to explain that, consider the following;

Two separated bar magnets will accelerate toward one another due to the magnetic force, if there is no friction to keep them apart. If you draw a circle around either magnet alone;

Where is an equal amount of momentum crossing that boundary, going in the "opposite" direction relative to the motion of each magnet?

The electromagnetic field is mediated by photons, so;

Why does this situation exert a force many orders of magnitude greater than a photon rocket?

The photons that are exchanged between the two magnets causes them to "attract" not repel, so conventional "logic" would say they are conveying the momentum in the wrong direction.

To understand this, you must consider not only the Amplitude of the field, but also the Phase, North vs South. This is where everyone is missing the mark, like Greg Egan using time averaged sin and cos, ignores the relative instantaneous phase and resulting interference pattern of the superposition.

Todd



Offline phaseshift

  • Full Member
  • *
  • Posts: 104
  • Seattle, WA
  • Liked: 84
  • Likes Given: 97
In conventional physics, it is clear that the momentum of the radiation injected into the cavity produces a back reaction upon the source of that radiation. Therefore if you put a box around [source + cavity], where the source includes a portable power supply (e.g. battery) - then the net momentum of the system is zero forever as viewed from outside the box.

If there is a radiation leak, thrust no greater than P/c ("equivalent photon rocket") may result from such a system, including thermal effects originating from the input power P.

Clearly here the claim being made is not conventional physics because:

1. The claim is made that the box as a whole will move
and
2. The claim is made that the magnitude of the thrust causing this movement exceeds the maximum expected thrust of the equivalent photon rocket by orders of magnitude.

Thus any successful attempt at explanation will not use conventional physics.


There is a corollary to all this:

If you attempt to explain this effect with conventional physics alone, you have made an error.

Thank you, thank  you, thank you.  Exactly.

But we've covered all of this literally dozens of times up thread.  Pretty please, don't rehash what we've already covered ad nauseum, and instead focus on what we haven't covered so that the thread doesn't loose meaning or focus.  Every few weeks this thread has a collective breather when someone yells "Unicorns don't exist!" then the lurkers ask a bunch of newbie questions, and then we go back to researching while the audience listens quietly. :)

Agreed! Yes, we keep going in circles rehashing the same stuff over and over - and every time that an attempt is made to go outside of the dogmatic circle the thread is jerked back 'into place'. There are a few that manage to keep bringing their ideas up for discussion (thank you Todd and others) but the ideas get lost in all the 1/4 wavelength vs 1/2 wavelength kind of discussions.  The answers lie outside the circle and it is certainly constructive to have such discussions. :)

We need a phase shift in thinking ;)
« Last Edit: 06/02/2015 08:46 pm by phaseshift »
"It doesn't have to be a brain storm, a drizzle will often do" - phaseshift

Offline SeeShells

  • Senior Member
  • *****
  • Posts: 2442
  • Every action there's a reaction we try to grasp.
  • United States
  • Liked: 3186
  • Likes Given: 2708
But is that a closed or open system.

Face the unfortunate fact that until more data is received from experiments conducted at a high level this thread could be doomed to go in circles.


Yes, that can be a consequence, or you can jump in with the experimenters and focus on the experiments in the hope that more experimental results will break the rhetoric cycle.
I need to remind myself that this is a Tar Baby Frustum and really all anyone here wants is to see it work. I've read and understood more about one EM Frustum than about anything I've ever done. I try to understand everyones input no matter how off center it may be or simplistic (like. maybe. could. we fill it with jello?) questions.
I love the passion and drive and creativity and so we rehash and rehash, explain a little differently. One day the light will go on and there will be an EUREKA moment "It works and we can make it better, or Damn it was just swamp gas all along".

Offline phaseshift

  • Full Member
  • *
  • Posts: 104
  • Seattle, WA
  • Liked: 84
  • Likes Given: 97
Is there anything that can be gained from looking at Shawyer's superconducting frustum experiment?  That has hardly been mentioned at all.  This is his 2nd generation thruster (TheTraveller correct me if its not) that he claims is far more powerful and he has gone to a squared off frustum.  Is this simply for ease of doing some of the math? Is it because working with superconductors is too difficult in a conical frustum? I could come up with a dozen more questions.

Thoughts?

Any ideas about dimensions?

« Last Edit: 06/02/2015 09:40 pm by phaseshift »
"It doesn't have to be a brain storm, a drizzle will often do" - phaseshift

Offline FutureStormtrooper

  • Member
  • Posts: 9
  • United States
  • Liked: 6
  • Likes Given: 52
I need to remind myself that this is a Tar Baby Frustum and really all anyone here wants is to see it work. I've read and understood more about one EM Frustum than about anything I've ever done. I try to understand everyones input no matter how off center it may be or simplistic (like. maybe. could. we fill it with jello?) questions.
I love the passion and drive and creativity and so we rehash and rehash, explain a little differently. One day the light will go on and there will be an EUREKA moment "It works and we can make it better, or Damn it was just swamp gas all along".

Does anyone know, off the top of their head, the refractive index of jello? Perhaps this hypothesis is worth investigating... worst that happens is somebody gets scalded by molten sugar.

Offline phaseshift

  • Full Member
  • *
  • Posts: 104
  • Seattle, WA
  • Liked: 84
  • Likes Given: 97


Does anyone know, off the top of their head, the refractive index of jello? Perhaps this hypothesis is worth investigating... worst that happens is somebody gets scalded by molten sugar.

http://www.instructables.com/id/Jello-Refraction-Experiment/step5/Calculating-the-Refractive-Index-of-Jello/
"It doesn't have to be a brain storm, a drizzle will often do" - phaseshift

Offline deltaMass

  • Full Member
  • ****
  • Posts: 955
  • A Brit in California
  • Liked: 671
  • Likes Given: 275
Quote from: WarpTech
K ~ exp[2GM/r*c^2]
So K ~ 1. What use, then, is it?

Uhmm.... (1/2)*Grad(K) = g/c^2  for example of using it to find the acceleration. What use is the metric tensor, g_uv?

It is an engineering tool because it is much easier working with K than it is working with g_uv and Einstein's equations, and it can be expressed in a frequency & bandwidth dependent way very easily. :)

I updated my previous post on this topic. Check it out.
http://forum.nasaspaceflight.com/index.php?topic=37642.msg1383566#msg1383566
I do believe you're missing my point, which is to point out that the magnitude of your 'K' is as close to unity as makes no difference.

Offline Flyby

  • Full Member
  • ***
  • Posts: 388
  • Belgium
  • Liked: 451
  • Likes Given: 48


Simply put, it moves because inside there is an EM stress energy tensor that is skewed to one side by constructive and destructive interference with the source. It would have a lot more thrust if the balance were 50/50, but as it is, only a small percentage of the waves walk out of phase to cause the interference. So the standing waves dominate, and there is very little thrust. The percentage results in higher NET thrust for higher Q, but it would be a more efficient thruster if the percentage were higher and the Q were lower.

Todd

Very interesting thesis there , Todd...
Assuming this is indeed the correct 50/50 proportion, wouldn't it be possible - for a mathematical skilled person - to calculated the frustum sidewall angle, for a given Q value?

Is there a way to calculate the amount of attenuation depending on the cone angle and then balance that against the energy stored, based upon the Q value?

It would be so great if there was a way to calculate the optimal cone angle in relation to the Q, in order to achieve optimal performance. That would be THE golden engineering tool for designing an EMdrive....

« Last Edit: 06/02/2015 09:36 pm by Flyby »

Offline WarpTech

  • Full Member
  • ****
  • Posts: 1407
  • Do it!
  • Statesville, NC
  • Liked: 1453
  • Likes Given: 1925
Quote from: WarpTech
K ~ exp[2GM/r*c^2]
So K ~ 1. What use, then, is it?

Uhmm.... (1/2)*Grad(K) = g/c^2  for example of using it to find the acceleration. What use is the metric tensor, g_uv?

It is an engineering tool because it is much easier working with K than it is working with g_uv and Einstein's equations, and it can be expressed in a frequency & bandwidth dependent way very easily. :)

I updated my previous post on this topic. Check it out.
http://forum.nasaspaceflight.com/index.php?topic=37642.msg1383566#msg1383566
I do believe you're missing my point, which is to point out that the magnitude of your 'K' is as close to unity as makes no difference.

The value is irrelevant. The acceleration is determined by the gradient derivative, wrt the coordinates. As you said, K~1, but the derivative at the surface of the earth yields an acceleration of 9.8 m/s^2, which is not irrelevant. The accelerated reference frame is not generated by K, but buy its gradient derivative.

« Last Edit: 06/02/2015 09:43 pm by WarpTech »

Offline deltaMass

  • Full Member
  • ****
  • Posts: 955
  • A Brit in California
  • Liked: 671
  • Likes Given: 275
You recall we were discussing whether c could change in the absence of a material medium.  I came into this when you tossed a "c/K" out there. If K~1 (as we both agree), then this becomes a semantic null statement.


Offline wallofwolfstreet

  • Full Member
  • *
  • Posts: 165
  • Liked: 169
  • Likes Given: 436
Quote
Before I try to explain that, consider the following;

Two separated bar magnets will accelerate toward one another due to the magnetic force, if there is no friction to keep them apart. If you draw a circle around either magnet alone;

Where is an equal amount of momentum crossing that boundary, going in the "opposite" direction relative to the motion of each magnet?

There is none, nor should there be.  Momentum doesn't have to be conserved locally.  You simply chose the wrong boundary for you circle to differentiate between local and global.

Quote
The electromagnetic field is mediated by photons, so;

Why does this situation exert a force many orders of magnitude greater than a photon rocket?


Not all photons can contribute to force like a photon rocket, they must be radiative.  The field emanating from a bar magnet is near field/reactive, there is no photon rocket effect whatsoever.

Quote
The photons that are exchanged between the two magnets causes them to "attract" not repel, so conventional "logic" would say they are conveying the momentum in the wrong direction.

To understand this, you must consider not only the Amplitude of the field, but also the Phase, North vs South. This is where everyone is missing the mark, like Greg Egan using time averaged sin and cos, ignores the relative instantaneous phase and resulting interference pattern of the superposition.

Todd

You see, this is where I still don't understand where you are coming from.  To use your two magnets pulling towards one another analogy, the apparent gain in momentum from one magnet was exactly cancelled when we chose a larger, more inclusive boundary with which to perform our momentum accounting.  So with the emdrive moving forward, what is it in your theory that moves back?  Our is it that the whole universe is not a large enough circle with which to account for momentum, and thus momentum is gained in another dimension/exotic field?

At the end of the day, phase, superposition and interference are all classical effects.  Nothing special there.  So what circle do I need to draw around the EMdrive, and what fields do I need to consider, such that I end up with a momentum that is equal and opposite the physical momentum of the drive?

Offline WarpTech

  • Full Member
  • ****
  • Posts: 1407
  • Do it!
  • Statesville, NC
  • Liked: 1453
  • Likes Given: 1925
You recall we were discussing whether c could change in the absence of a material medium.  I came into this when you tossed a "c/K" out there. If K~1 (as we both agree), then this becomes a semantic null statement.

At the surface of the Earth, for the Earth's gravitational field;

K = 1/(1 - 2GM/r*c^2) ~ 1
Grad(K) = 2*g/c^2

Inside the frustum,

K_frustum = sqrt(1 + (a(z)/k)^2), where "a(z)" is the attenuation factor as a function of position along the "z" axis of the frustum, and "k" is the free space wave number. For the wavelength's inside the frustum;

K_frustum > 1, and
Grad(K_frustum) = (a/K)*(1/k^2)*da(z)/dz

It is only insignificant if the attenuation factor "a = 0" or if "a = constant". In a frustum it does not equal zero and it is not a constant.

I think you just like to antagonize! This is not THAT difficult to understand.

Todd



Offline Rodal

  • Senior Member
  • *****
  • Posts: 5911
  • USA
  • Liked: 6124
  • Likes Given: 5564
Concerning superposiiton, unlike a particle, a wave is able to explore simultaneously multiple paths while undergoing chaotic diffusion.  Typically after a characteristic time these multiple paths destructively interfere suppressing further diffusion. 

Offline WarpTech

  • Full Member
  • ****
  • Posts: 1407
  • Do it!
  • Statesville, NC
  • Liked: 1453
  • Likes Given: 1925
Quote
...

Where is an equal amount of momentum crossing that boundary, going in the "opposite" direction relative to the motion of each magnet?

There is none, nor should there be.  Momentum doesn't have to be conserved locally.  You simply chose the wrong boundary for you circle to differentiate between local and global.

So you're saying that a magnet interacting with a magnetic field does not conserve momentum? Of course it does!

Quote

Quote
The electromagnetic field is mediated by photons, so;

Why does this situation exert a force many orders of magnitude greater than a photon rocket?


Not all photons can contribute to force like a photon rocket, they must be radiative.  The field emanating from a bar magnet is near field/reactive, there is no photon rocket effect whatsoever.


Correct! The frustum is on order of 1/2 wavelength in diameter and length. It is definitely a "near-field" and such things can and should be applicable!

Quote

Quote
The photons that are exchanged between the two magnets causes them to "attract" not repel, so conventional "logic" would say they are conveying the momentum in the wrong direction.

To understand this, you must consider not only the Amplitude of the field, but also the Phase, North vs South. This is where everyone is missing the mark, like Greg Egan using time averaged sin and cos, ignores the relative instantaneous phase and resulting interference pattern of the superposition.

Todd

You see, this is where I still don't understand where you are coming from.  To use your two magnets pulling towards one another analogy, the apparent gain in momentum from one magnet was exactly cancelled when we chose a larger, more inclusive boundary with which to perform our momentum accounting.  So with the emdrive moving forward, what is it in your theory that moves back?  Our is it that the whole universe is not a large enough circle with which to account for momentum, and thus momentum is gained in another dimension/exotic field?

At the end of the day, phase, superposition and interference are all classical effects.  Nothing special there.  So what circle do I need to draw around the EMdrive, and what fields do I need to consider, such that I end up with a momentum that is equal and opposite the physical momentum of the drive?

Nothing is moving "backwards". Momentum is flowing INTO the boundary, NOT OUT of the boundary. It works both ways. Divergence is not zero when the source is turned on. The Poynting vector is not zero. The momentum put inside is then attenuated asymmetrically, and the resulting forces and amplitudes depend on the relative phase of the waves, not simply their time-averaged pressure. The conservation law says is that there must be divergence through the boundary, it can be in either direction, in or out. It isn't ONLY expelling something out that satisfies this condition.

Todd

Offline SeeShells

  • Senior Member
  • *****
  • Posts: 2442
  • Every action there's a reaction we try to grasp.
  • United States
  • Liked: 3186
  • Likes Given: 2708
You recall we were discussing whether c could change in the absence of a material medium.  I came into this when you tossed a "c/K" out there. If K~1 (as we both agree), then this becomes a semantic null statement.

At the surface of the Earth, for the Earth's gravitational field;

K = 1/(1 - 2GM/r*c^2) ~ 1
Grad(K) = 2*g/c^2

Inside the frustum,

K_frustum = sqrt(1 + (a(z)/k)^2), where "a(z)" is the attenuation factor as a function of position along the "z" axis of the frustum, and "k" is the free space wave number. For the wavelength's inside the frustum;

K_frustum > 1, and
Grad(K_frustum) = (a/K)*(1/k^2)*da(z)/dz

It is only insignificant if the attenuation factor "a = 0" or if "a = constant". In a frustum it does not equal zero and it is not a constant.

I think you just like to antagonize! This is not THAT difficult to understand.

Todd
For whatever reason he is questioning it's doing one thing, it's helping me to see better where your coming from. Thanks

Offline WarpTech

  • Full Member
  • ****
  • Posts: 1407
  • Do it!
  • Statesville, NC
  • Liked: 1453
  • Likes Given: 1925
"Scientists have argued for more than a century about the momentum of light in materials," Leonhardt told Phys.org. "Is it Abraham's, is it Minkowski's? We discovered that momentum is not a fundamental quantity, but it is made in the interplay between light and matter, and it depends on the ability of the light to move the material. If the medium does not move, it is Minkowski's, and if it moves, Abraham's. This was not understood before."

Read more at: http://phys.org/news/2015-06-physicists-pressure.html#jCp


Offline RotoSequence

  • Senior Member
  • *****
  • Posts: 2208
  • Liked: 2068
  • Likes Given: 1535
"Scientists have argued for more than a century about the momentum of light in materials," Leonhardt told Phys.org. "Is it Abraham's, is it Minkowski's? We discovered that momentum is not a fundamental quantity, but it is made in the interplay between light and matter, and it depends on the ability of the light to move the material. If the medium does not move, it is Minkowski's, and if it moves, Abraham's. This was not understood before."

Read more at: http://phys.org/news/2015-06-physicists-pressure.html#jCp

This looks strikingly familiar...

Tags:
 

Advertisement NovaTech
Advertisement Northrop Grumman
Advertisement
Advertisement Margaritaville Beach Resort South Padre Island
Advertisement Brady Kenniston
Advertisement NextSpaceflight
Advertisement Nathan Barker Photography
0